RegistrierenRegistrieren   LoginLogin   FAQFAQ    SuchenSuchen   
Spinwellen
 
Neue Frage »
Antworten »
    Foren-Übersicht -> Quantenphysik
Autor Nachricht
nekros7



Anmeldungsdatum: 18.08.2006
Beiträge: 154

Beitrag nekros7 Verfasst am: 26. Jan 2008 11:13    Titel: Spinwellen Antworten mit Zitat

Hallo!
Ich hätte eine Frage zu Ferromagnetismus. Wir hatten in der Vorlesung besprochen, dass es energetisch günstiger seien kann, statt einen ganzen Spin umzuklappen, die Anregungsenergie auf sämtliche Spins des Festkörpers zu übertragen. Das Zitat war "die Spins nur ein bisschen umklappen". Das hat mich doch sehr verwirrt, da ich dachte, dass die Eigenzustände von einem Spin nur Up oder Down seien. In gängigen Lehrbüchern ist immer die Rede von Dephasierung der Spins, da die Spins um die z-Achse präzedieren...Im Ibach-Lüth Lehrbuch bzw. Kopitzki konnte ich auch keine sinnvolle Erklärung finden. Da sind immer nur Bilder von Spin Ketten, die ihre z-Komponente von links nach recht verkleinern, aber nie den ganzen Spin am Ende umklappen.
Lange Geschichte....

Wie genau funktionieren Spinwellen?
Klappt am Ende ein Spin um oder nicht?

Nekros7
dermarkus
Administrator


Anmeldungsdatum: 12.01.2006
Beiträge: 14788

Beitrag dermarkus Verfasst am: 26. Jan 2008 12:57    Titel: Antworten mit Zitat

Spins können sehr wohl Zustände "dazwischen" annehmen.

Kennst du schon den Unterschied zwischen dem Erwartungswert einer Messung und den Werten, auf den ein Zustand bei einer Messung projiziert werden kann?
nekros7



Anmeldungsdatum: 18.08.2006
Beiträge: 154

Beitrag nekros7 Verfasst am: 26. Jan 2008 13:48    Titel: Antworten mit Zitat

Der Erwartungswert gibt an, was mein wahrscheinlichstes Messergebnis ist. Und da beim Spin die dazugehörige Observable die 2 komponentigen Pauli Spin Matrizen sind, dachte ich dass ich entweder +1/2 hquer oder -1/2 hquer bekommen kann.
Oder habe ich eine falsche Vorstellung?
dermarkus
Administrator


Anmeldungsdatum: 12.01.2006
Beiträge: 14788

Beitrag dermarkus Verfasst am: 26. Jan 2008 23:10    Titel: Antworten mit Zitat

nekros7 hat Folgendes geschrieben:
Der Erwartungswert gibt an, was mein wahrscheinlichstes Messergebnis ist.

Diese Formulierung stimmt nur manchmal. (Nämlich nur für kontinuierliche, nicht für gequantelte Messvariablen, siehe Beispiel unten.)

Zitat:

Und da beim Spin die dazugehörige Observable die 2 komponentigen Pauli Spin Matrizen sind, dachte ich dass ich entweder +1/2 hquer oder -1/2 hquer bekommen kann.
Oder habe ich eine falsche Vorstellung?

Ich bin einverstanden, dass man, wenn man die Komponente eines Spins in einer bestimmten Raumrichtung (sagen wir zum Beispiel mal, die z-Komponente des Spins eines Teilchens mit Spin 1/2) misst, bei jeder Messung entweder oder als Messergebnis der Einzelmessung erhalten wird. Es gibt also nur zwei mögliche Messwerte, die als Ergebnis dieser Einzelmessung herauskommen können.

Der Erwartungswert muss allerdings kein möglicher Messwert einer Einzelmessung sein, denn er ist der Mittelwert der Einzelmesswerte, die man erhält, wenn man am selben Zustand viele Male dieselbe Einzelmessung vornimmt.

Nimm dir mal einfach als ganz klassisches Beispiel einen Würfel. Der hat nur sechs mögliche Messwerte, nämlich 1, 2, 3, 4, 5 und 6. Aber sein Erwartungswert (wenn er nicht gezinkt ist, also wenn alle Zahlen mit derselben Wahrscheinlichkeit auftreten) ist 3,5 (drei komma fünf).
nekros7



Anmeldungsdatum: 18.08.2006
Beiträge: 154

Beitrag nekros7 Verfasst am: 27. Jan 2008 10:45    Titel: Antworten mit Zitat

Ok.
Wie ist das dann bei Spinwellen? Klappt jetzt ein Spin von +1/2 auf -1/2 um oder nicht? Wie kann man sich Spinwellen vorstellen?
dermarkus
Administrator


Anmeldungsdatum: 12.01.2006
Beiträge: 14788

Beitrag dermarkus Verfasst am: 27. Jan 2008 13:11    Titel: Antworten mit Zitat

nekros7 hat Folgendes geschrieben:
Klappt jetzt ein Spin von +1/2 auf -1/2 um oder nicht?

Ja und nein.

Wenn du Einzelmessungen machst und zum Beispiel zu verschiedenen Zeiten die Komponente eines Spins entlang der z-Richtung misst, dann wirst immer entweder feststellen, dass die Komponente gleichgeblieben ist oder dass sie von 1/2 zu minus 1/2 bzw. umgekehrt umgeklappt ist.

Wenn du aber an ein und demselben (also immer genau gleich präparierten) Spinzustand dieselbe solche Einzelmessung vornimmst, dann wirst du feststellen, dass die Verteilung der Messwerte und damit der der Mittelwert all dieser Messungen davon abhängt, wie "schräg" der Spinpfeil in dem präparierten Zustand steht.

Auf die z-Achse bezogen, nennt sich so ein "schräger" Zustand dann zum Beispiel eine kohärente Überlagerung der Zustände +1/2 und -1/2.
nekros7



Anmeldungsdatum: 18.08.2006
Beiträge: 154

Beitrag nekros7 Verfasst am: 27. Jan 2008 15:54    Titel: Antworten mit Zitat

So ganz verstehe ich das noch nicht. In der Vorlesung hatten wir ein Bild von einer Spinkette. Die Spins drehen sich alle um die z-Achse und zwar phasenverschoben. Aber keiner von den Spins klappt um. Es heisst: "Spinumklapp gleichmaessig verteilt auf alle spins"
Wie hat man das zu verstehen?
dermarkus
Administrator


Anmeldungsdatum: 12.01.2006
Beiträge: 14788

Beitrag dermarkus Verfasst am: 27. Jan 2008 22:22    Titel: Antworten mit Zitat

Hm, magst du mal selbst versuchen, das in das bisher gesagte und bekannte einzuordnen?

Wenn du dir mal vorstellen möchtest, dass die Spins tatsächlich schräg da drin stehen, in welcher Richtung muss man dann die Spinkomponente eines solchen Beispielspins messen, um immer dasselbe Ergebnis zu erhalten?

Kann man die Umklappenergie, die man für das Umklappen eines einzigen Spins benötigen würde, statt dessen auch dafür verwenden, mehrere Spins ein bisschen schräg zu stellen?

Spart man dabei sogar Energie, wenn man die Wechselwirkung der Spin-Magnetfelder untereinander dabei berücksichtigt?
nekros7



Anmeldungsdatum: 18.08.2006
Beiträge: 154

Beitrag nekros7 Verfasst am: 29. Jan 2008 10:26    Titel: Antworten mit Zitat

Also, um immer den gleichen wert fuer den spin zu bekommen, wuerde ich sagen entlang der z-achse. Aber die aendert sich doch nicht, wenn sich der spin etwas phasenverschoben gegenueber einem anderen ist. Die aendert sich doch nur wenn der spin sich etwas nach unten neigt. Das habe ich aber in den Bildern aus den FK-Buechern nicht gesehen.

Also ist es so zu verstehen, dass sich der Spin nur ein bisschen neigt und die z-Komponente kleiner wird und somit es energetisch guenstiger ist, alle spins etwas zu neigen, anstatt einen ganzen umzuklappen.

Was hat das jetzt auf sich mit den Spinmagnetfelder? Sind damit die magnetischen Momente der Spins gemeint?
dermarkus
Administrator


Anmeldungsdatum: 12.01.2006
Beiträge: 14788

Beitrag dermarkus Verfasst am: 29. Jan 2008 16:03    Titel: Antworten mit Zitat

Ja, mit "Spinmagnetfelder" habe ich die Magnetfelder gemeint, die durch die magnetischen Momente der Spins erzeugt werden. Denn die Spins sind ja nichts anderes als winzige Minimagnete.

Ich stelle mir also im klassischen Bild die Spins tatsächlich so geneigt wie kleine Minimagnete vor, deren Orientierung im Raum sich entlang der Ausbreitungsrichtung der Spinwelle periodisch verändert.

Sagen dir die Begriffe "Bloch-Vektor" und "Bloch-Kugel" schon etwas? Kennst du schon die sogenannten Rabi-Oszillationen?

---------------------------------------------------

Um die Frage der Spinorientierung, insbesondere dabei die Frage der Neigung des Spins relativ zur z-Achse, mit deinem Wissen über quantenmechanisch gequantelte Eigenwerte von Messoperatoren stimmig unter einen Hut zu bringen, dürfte es sich für dich vor allem lohnen, mal die quantenmechanische Seite genauer unter die Lupe zu nehmen:

Stell dir vor, ein einzelner Spin sei so präpariert, dass er in Richtung der positiven x-Achse zeigt. Also so, dass eine Messung seiner Spinkomponente bezüglich der x-Achse mit 100% Wahrscheinlichkeit den Wert ergibt.

* Welche Messwerte treten dann mit welcher Wahrscheinlichkeit auf, wenn man an diesem Spin eine Messung durchführt, die die Spinkomponente entlang der z-Achse misst?

* Und welche Messwerte treten mit welcher Wahrscheinlichkeit auf, wenn man an diesem Spin eine Messung durchführt, die die Spinkomponente entlang der Richtung der y-z-Winkelhalbierenden misst?
nekros7



Anmeldungsdatum: 18.08.2006
Beiträge: 154

Beitrag nekros7 Verfasst am: 04. Feb 2008 09:47    Titel: Antworten mit Zitat

Zitat:
* Welche Messwerte treten dann mit welcher Wahrscheinlichkeit auf, wenn man an diesem Spin eine Messung durchführt, die die Spinkomponente entlang der z-Achse misst?


Dann ist es doch nicht moeglich, eine Komponente entlang der z-achse zu messen, da der Spin entlang der x-Achse polarisiert ist. Meine ausgezeichnete Achse ist in diesem Fall die x-Achse. Und ich kann ja nur [S^2,S_3] gleichzeitig scharf messen.

Zitat:


Und welche Messwerte treten mit welcher Wahrscheinlichkeit auf, wenn man an diesem Spin eine Messung durchführt, die die Spinkomponente entlang der Richtung der y-z-Winkelhalbierenden misst?

keine Ahnung, ich wuerde sagen, dass man da nicht messen kann, was mit Sicherheit nicht stimmt. Ich dachte mit den Pauli Spin Matrizen kann man nur entlang einer fest vorgegebenen Ahcse (x,y,z) messen.

QM ist ziemlich hart. Bin gerade dabei alles fuer das Diplom zu wiederholen. Man merkt dabei, dass man theo physik nicht leicht auf das Experiment anwenden kann.
[/quote]
dermarkus
Administrator


Anmeldungsdatum: 12.01.2006
Beiträge: 14788

Beitrag dermarkus Verfasst am: 04. Feb 2008 14:52    Titel: Antworten mit Zitat

Oh, da unterscheidest du noch nicht klar genug zwischen dem, was du unter "kann man messen" und "kann man scharf messen" zu verstehen scheinst.

Messen kann man die z-Komponente eines Spins 1/2 mit Spin_x = +1/2 sehr wohl. Bei dieser Einzelmessung bekommt man als Ergebnis einen der beiden Werte Spin_z = +1/2 oder -1/2, denn das sind ja die einzigen beiden möglichen Messwerte einer Einzelmessung. Führt man diese Einzelmessung allerdings sehr oft wiederholt an einem immer wieder gleich präparierten Zustand durch, dann wird man feststellen, dass man in 50% der Fälle Spin_z=+1/2 misst, und in 50% der Fälle Spin_z = -1/2.

Messen kann man das also prima, es kommt halt nur nicht immer derselbe Messwert heraus.

Wenn du mit "scharf messen können" die Forderung meinst, dass bei einer solchen Einzelmessung an einem Zustand immer genau dasselbe Ergebnis herauskommen soll, dann (und nur dann Augenzwinkern ) muss der präparierte Zustand dafür ein Eigenzustand des Messoperators sein. Das wäre zum Beispiel dann der Fall, wenn du einen Zustand mit Spin_z = +1/2 präparierst und daran die Spinkomponente in z-Richtung misst.

Wird es damit schon klarer, und kannst du damit die Fragen von eben besser beantworten?
nekros7



Anmeldungsdatum: 18.08.2006
Beiträge: 154

Beitrag nekros7 Verfasst am: 08. Feb 2008 12:27    Titel: Antworten mit Zitat

Ok. Wenn meine yz-Achse meine vorgegebene achse ist, dann kann ich die projektion auf die achse messen. Das heisst mit 100% Spin +1/2, da der spin in die pos. x-achse zeigt.
dermarkus
Administrator


Anmeldungsdatum: 12.01.2006
Beiträge: 14788

Beitrag dermarkus Verfasst am: 08. Feb 2008 13:19    Titel: Antworten mit Zitat

Überleg lieber noch mal, oder mach dir gerne eine kleine Skizze:

Welchen Winkel bilden die y-z-Winkelhalbierende und die x-Achse miteinander?
nekros7



Anmeldungsdatum: 18.08.2006
Beiträge: 154

Beitrag nekros7 Verfasst am: 10. Feb 2008 12:42    Titel: Antworten mit Zitat

Also der Winkel ist 90 Grad. Ist es dann so, da der Spin sich um die positive x-achse dreht, die Projektion 0 gibt und zwar mit 100% Wahrscheinlichkeit?
Wenn das so ist, dann ist der Spin aber parallel zu x-achse. Ich dachte, dass der auf einen Kegel praezediert.
dermarkus
Administrator


Anmeldungsdatum: 12.01.2006
Beiträge: 14788

Beitrag dermarkus Verfasst am: 11. Feb 2008 18:46    Titel: Antworten mit Zitat

nekros7 hat Folgendes geschrieben:
Also der Winkel ist 90 Grad. die Projektion 0 gibt und zwar mit 100% Wahrscheinlichkeit?

Einverstanden, der Winkel ist dann 90°, und die klassische Projektion des Spinvektors auf diese Achse würde 0 ergeben.

Nun ist die Messung des Spins aber natürlich keine klassische Projektion. Sondern eine quantenmechanische Messung. Kann bei einer solchen Einzelmessung also überhaupt der Messwert Null auftreten? Welche Messwerte können hier auftreten, und mit welcher Wahrscheinlichkeit treten sie auf?

Zitat:

Ist es dann so, da der Spin sich um die positive x-achse dreht,

Wenn das so ist, dann ist der Spin aber parallel zu x-achse. Ich dachte, dass der auf einen Kegel praezediert.

Vom Präzedieren von Spins ist hier erst einmal noch überhaupt nicht die Rede. Denn damit ein Spin präzediert, braucht er ja erst einmal ein Magnetfeld, dessen Richtung und Stärke bestimmt, um welche Achse und mit welcher Winkelgeschwindigkeit er präzedieren soll.

Die Frage, auf welche räumliche Bezugsachse sich die Messung der Komponente eines Spins beziehen soll, lässt sich natürlich auch in kompletter Abwesenheit von äußeren Magnetfeldern betrachten.
nekros7



Anmeldungsdatum: 18.08.2006
Beiträge: 154

Beitrag nekros7 Verfasst am: 11. Feb 2008 20:43    Titel: Antworten mit Zitat

Als Messwerte koennen ja nur die Eigenwerte in Frage kommen. Also muesste mit 50% Wahrscheinlichkeit +1/2 und mit 50% Wahrscheinlichkeit -1/2 herauskommen, so dass sich im Mittel 0 ergibt.
dermarkus
Administrator


Anmeldungsdatum: 12.01.2006
Beiträge: 14788

Beitrag dermarkus Verfasst am: 12. Feb 2008 00:36    Titel: Antworten mit Zitat

Einverstanden smile mit 50% Wahrscheinlichkeit und mit ebenfalls 50% Wahrscheinlichkeit.

Was würdest du mit dem, was du bisher verstanden hast, schätzen:

Wenn du für einem in x-Richtung ausgerichteten Spin 1/2 nun die Komponente in x-y-Richtung messen würdest, welche Werte würden dann dabei mit ungefähr welcher Wahrscheinlichkeit auftreten?
nekros7



Anmeldungsdatum: 18.08.2006
Beiträge: 154

Beitrag nekros7 Verfasst am: 12. Feb 2008 09:33    Titel: Antworten mit Zitat

Na ja, ist das ergebnis auf alle faelle nicht 0. Kann man sich das ganze wie Vektoraddition vorstellen, das heisst, dass wir mit 50% wahtrscheinlichkeit s_x=1/2 hquer und mit 50% Wahrscheinlichkeit s_y=1/2 hquer herausbekommen. Dann adieren sich die beiden Komponenten zu xy-Gerade.
dermarkus
Administrator


Anmeldungsdatum: 12.01.2006
Beiträge: 14788

Beitrag dermarkus Verfasst am: 12. Feb 2008 11:24    Titel: Antworten mit Zitat

Schau mal genauer hin:

Es geht hier nicht um eine vektorielle Addition von - Spinkomponente und -Spinkomponente zu einem Vektor, der in xy-Richtung zeigt.

Die Frage war, mit welcher Wahrscheinlichkeit werden die verschiedenen möglichen Werte der Komponente des Spins entlang der xy-Winkelhalierenden gemessen. Wenn der Spin 1/2 in x-Richtung zeigt.
nekros7



Anmeldungsdatum: 18.08.2006
Beiträge: 154

Beitrag nekros7 Verfasst am: 12. Feb 2008 12:11    Titel: Antworten mit Zitat

Es gibt doch nur zwei Werte fuer die Winkelhalbierende, einmal die in positiver xy-Ebene, und einmal in negativer xy-Ebene. Da der Spin in positiver x-Richtung zeigt, muessen sich die Eigenwerte fuer die negative xy-Richtung rausmitteln und die die fuer die positive xy-Ebene muesste 100% +1/2 hquer geben.
Anscheinend stehe ich gewaltig auf dem Schlauch.
dermarkus
Administrator


Anmeldungsdatum: 12.01.2006
Beiträge: 14788

Beitrag dermarkus Verfasst am: 12. Feb 2008 12:15    Titel: Antworten mit Zitat

Mal anders gefragt: In welche Richtung zeigend müsste denn der Spin 1/2 präpariert werden, damit der Messwert mit 100 % Wahrscheinlichkeit auftritt?
nekros7



Anmeldungsdatum: 18.08.2006
Beiträge: 154

Beitrag nekros7 Verfasst am: 12. Feb 2008 15:05    Titel: Antworten mit Zitat

In xy-Richtung. Dann bekommt man zu 100% 1/2 hquer.
dermarkus
Administrator


Anmeldungsdatum: 12.01.2006
Beiträge: 14788

Beitrag dermarkus Verfasst am: 12. Feb 2008 15:18    Titel: Antworten mit Zitat

Einverstanden.

Wie verändert sich im Vergleich dazu diese Wahrscheinlichkeit, +1/2 zu messen, wenn der Spin statt dessen etwas schräg zur Bezugsachse (also der xy-Achse) steht? Und wie verändert sich die Wahrscheinlichkeit, -1/2 zu messen?
nekros7



Anmeldungsdatum: 18.08.2006
Beiträge: 154

Beitrag nekros7 Verfasst am: 12. Feb 2008 15:28    Titel: Antworten mit Zitat

Wenn der Spin etwas schraeg zu xy-Achse steht, dann bekommt man nicht mehr ganz mit 100% den Messwert +1/2 hquer. Also zum Beispiel dann mit 95%. Die restlichen 5% muessten dann -1/2hquer ergeben.
dermarkus
Administrator


Anmeldungsdatum: 12.01.2006
Beiträge: 14788

Beitrag dermarkus Verfasst am: 12. Feb 2008 15:48    Titel: Antworten mit Zitat

Einverstanden smile

Ob das nun ganz genau 95% sind oder etwa 75% oder irgendein anderer Wert zwischen 50% und 100%, das soll uns hier erst einmal egal sein.

Als wichtiges Ergebnis unserer Überlegungen würde ich vorschlagen, festzuhalten:

* Die Richtung, in die der Spinpfeil zeigt,

* die Richtung, entlang der die Komponente des Spins zu einem bestimmten Zeitpunkt gemessen wird

* und die Richtung, in die ein etwaiges äußeres Magnetfeld zeigt

sind zunächst alle drei komplett voneinander unabhängige Dinge. Damit kann ein Spin auch problemlos "wirklich" schräg stehen, sowohl schräg in Bezug auf eine für eine Messung gewählte Bezugsachse, als auch schräg in Bezug auf die Richtung eines eventuellen äußeren Magnetfeldes.

Klärt das schon den wichtigsten Teil deiner anfänglichen Frage?
nekros7



Anmeldungsdatum: 18.08.2006
Beiträge: 154

Beitrag nekros7 Verfasst am: 12. Feb 2008 16:11    Titel: Antworten mit Zitat

Ja. Jetzt wird es schon klarer. Mein groesster Denkfehler war, dass ich angenommen hatte, dass der Spin nur zwei Einstellungen hat. In Wahrheit hat er mehr, nur falls ich messe, entlang einer bestimmten Achse, an die sich der Spin ausgerichtet hat, wird sich im Mittel +1/2 hquer bzw. -1/2 hquer geben. Messe ich entlang einer orthogonalen Achse, erhalte ich auch +1/2 hquer und -1/2 hquer, aber im Mittel 0.

Habe ich das so richtig verstanden?
dermarkus
Administrator


Anmeldungsdatum: 12.01.2006
Beiträge: 14788

Beitrag dermarkus Verfasst am: 12. Feb 2008 16:29    Titel: Antworten mit Zitat

Hm, magst du das am besten einfach nochmal deutlich genauer und ausführlicher aufschreiben? Und sicherstellen, dass keine versehentlichen Fehler drin sind, die den Sinn verdrehen?

Denn ich habe die Vermutung, du bist mit dem Verstehen schon auf dem richtigen Weg, und hast einfach mindestens teilweise noch nicht so ganz sorgfältig genug Gedanken geordnet und formuliert, als du das eben geschrieben hast.
nekros7



Anmeldungsdatum: 18.08.2006
Beiträge: 154

Beitrag nekros7 Verfasst am: 12. Feb 2008 16:48    Titel: Antworten mit Zitat

Was ich meinte war, wenn ich einen Spin habe der entlang der x-achse polarisiert ist und ich messe entlang z.B. der zy-Winkelhalbierenden, dann bekomme ich die Eigenwerte +1/2 hquer (50%) und -1/2 hquer (50%). Das heisst im Mittel 0.
Weiterhin habe ich immer gedacht, dass Spins entweder +1/2 hquer oder -1/2 hquer haben koennen. Das gilt aber nur fuer die Messung. In Wahrheit (das heisst, wenn ich das System nicht anschaue, also keine Wechselwirkung mit einem "Messgeraet" habe) koennen die Spins sich schon ein wenig verkippen.
Ich vermute, dass dann so die Spinwellen zustande kommen, dass sich tatsaechlich die Spins leicht verkippen. Ich habe immer gedacht, dass spins nur up oder down seien koennen.

Sind meine Gedanken auf dem richtigen Weg?
dermarkus
Administrator


Anmeldungsdatum: 12.01.2006
Beiträge: 14788

Beitrag dermarkus Verfasst am: 12. Feb 2008 17:13    Titel: Antworten mit Zitat

nekros7 hat Folgendes geschrieben:
Was ich meinte war, wenn ich einen Spin habe der entlang der x-achse polarisiert ist und ich messe entlang z.B. der zy-Winkelhalbierenden, dann bekomme ich die Eigenwerte +1/2 hquer (50%) und -1/2 hquer (50%). Das heisst im Mittel 0.
Weiterhin habe ich immer gedacht, dass Spins entweder +1/2 hquer oder -1/2 hquer haben koennen. Das gilt aber nur fuer die Messung. In Wahrheit (das heisst, wenn ich das System nicht anschaue, also keine Wechselwirkung mit einem "Messgeraet" habe) koennen die Spins sich schon ein wenig verkippen.
Ich vermute, dass dann so die Spinwellen zustande kommen, dass sich tatsaechlich die Spins leicht verkippen. Ich habe immer gedacht, dass spins nur up oder down seien koennen.

Sind meine Gedanken auf dem richtigen Weg?

Ja, damit bin ich komplett einverstanden smile

Hast du auch schon verstanden, was mich alles an deinen Formulierungen im vorangegangenen Beitrag gestört haben könnte?

nekros7 hat Folgendes geschrieben:
dass der Spin nur zwei Einstellungen hat. In Wahrheit hat er mehr,

vor allem muss man in Wahrheit erstmal genauer hinschauen, auf was sich das mit den zwei Einstellungen bezieht.

Ein Spin 1/2 kann beliebig viele Einstellungen im Raum haben, er kann in jede beliebige Raumrichtung zeigen.

Führt man eine Messung bezüglich einer bestimmten Bezugsachse durch, so erhält man bei einer solchen Messung immer nur einen von zwei möglichen Werten. Dies sind die beiden möglichen Einstellungen eines Spins 1/2, von denen in der Quantenmechanik so oft die Rede ist, dass man gar nicht mehr jedes Mal dazusagt, auf was genau man sich damit bezieht.

Liegt ein äußeres Magnetfeld an, dann wählt man so oft die Bezugsachse für eine Messung auch tatsächlich in Magnetfeldrichtung (weil dann praktischerweise der Kippwinkel der Spins zur Bezugsachse auch während der Präzession konstant bleibt), dass viele leicht vergessen, dass die Festlegung der Achse für die Messung einer Spinkomponente auch völlig unabhängig vom Vorhandensein eines Magnetfeldes gewählt werden kann.

Zitat:

nur falls ich messe, entlang einer bestimmten Achse, an die sich der Spin ausgerichtet hat, wird sich im Mittel +1/2 hquer bzw. -1/2 hquer geben.

Gehe ich recht in der Annahme, dass das "im Mittel" hier einfach nur ein grober Tippfehler oder ein Flüchtigkeits-Denkfehler war?

Zitat:

Messe ich entlang einer orthogonalen Achse, erhalte ich auch +1/2 hquer und -1/2 hquer, aber im Mittel 0.

Ich weiß, was du meinst, aber eigentlich musst du dazusagen, bezüglich was die Achse orthogonal sein soll (orthogonal zur Richtung des Spin-Vektors), sonst ist diese Aussage unvollständig.
nekros7



Anmeldungsdatum: 18.08.2006
Beiträge: 154

Beitrag nekros7 Verfasst am: 12. Feb 2008 17:25    Titel: Antworten mit Zitat

Ja das war ein verdammt dummer Tippfehler....das mit der Orthogonalitaet war auch nicht ganz eindeutig.
Nochmals zu den Spinwellen: Wie genau hat man diese Bilder von den Spinwellen in den FK Buechern z.B Kopitzki oder Kittel zu verstehen? Man sieht da so 10 Spins nebeneinander, die alle die gleiche Verkippung haben, nur phasenverschoben. Was hat diese Phasenverschiebung zu bedeuten?
In der Vorlesung hatten wir naemlich, dass bei einer Spinwelle, sich alle Spins etwas verkippen. In den Bildern sind die aber alle gleich geneigt.
dermarkus
Administrator


Anmeldungsdatum: 12.01.2006
Beiträge: 14788

Beitrag dermarkus Verfasst am: 12. Feb 2008 17:33    Titel: Antworten mit Zitat

Das passt beides zusammen: Stell dir alle Spins gekippt vor, die Frage ist dabei nur, ob sie dabei und während dem Präzedieren alle in die gleiche oder in unterschiedliche Richtungen zeigen.

Weil die Spins phasenverschoben präzedieren, zeigen sie nicht immer alle in dieselben Richtung (wie das bei einer "synchronen" Präzession ohne Phasenverschiebung der Fall wäre), sondern sind gegeneinander verkippt.

Und wenn zwei Spins gegeneinander verkippt sind, dann hat der eine im Magnetfeld des anderen eine größere Energie (ebenso der andere im Magnetfeld des einen), als wenn beide parallel zueinander ausgerichtet wären.

Und eben diese Energiezunahme durch Verkippen der Spins gegeneinander ist die Anregungsenergie der Spinwellen.
nekros7



Anmeldungsdatum: 18.08.2006
Beiträge: 154

Beitrag nekros7 Verfasst am: 12. Feb 2008 17:55    Titel: Antworten mit Zitat

Achso, also die Verkippung ist gegen den "Nachbar"Spin zu verstehen. Ok, jetzt habe ich das alles verstanden.
Was mich aber am meisten freut, dass ich den kapitalen Denkfehler mit den Spins, dass die nur in zwei Richtungen zeigen koennen, endlich loshabe.
Vielen Dank fuer deine Geduld
Neue Frage »
Antworten »
    Foren-Übersicht -> Quantenphysik